Solving Improper Integrals: 1/(3∙√x)dx

  • Thread starter Thread starter mirandasatterley
  • Start date Start date
  • Tags Tags
    Integrals
Click For Summary
The discussion centers on evaluating the improper integral of 1/(3√x) from 0 to infinity. The integral is transformed into a limit due to the infinite upper bound and the undefined lower bound at zero. The antiderivative is found to be x^(2/3), which diverges as x approaches infinity, while it approaches zero as x approaches zero. The conclusion is that the integral diverges because the result is infinity. The problem specifically asks for an explanation of the divergence, which is confirmed by the behavior of the antiderivative at the limits.
mirandasatterley
Messages
61
Reaction score
0

Homework Statement



Evaluate the following improper integrals of explain why they don't converge.
Integral from 0 to infinity(1/the cubed root of x)dx
I'm not sure how to make forulas, so this is the best I can do:
0∫∞ (1/(3∙√x))dx

Homework Equations



No equations

The Attempt at a Solution



I know that when there is ∞ as an upper bound, the intergration is changed to:

lim as b→∞ 0∫b (1/(3∙√x))dx
But in this form, the 0 is a problem.

and if the lower bound, 0, causes the function to be undefined, the integration is changed to:

lim as a→0+ a∫∞ (1/(3∙√x))dx
But, in this for the infinity is still a problem.


Is there any way to combine the two so I can solve this.
Any help is appreciated.
 
Physics news on Phys.org
Well, the first thing you had better do is actually write out the anti- derivative!
What is \int \frac{1}{^3\sqrt{x}}dx= \int x^{-\frac{1}{3}}dx?

Does it converge as x goes to 0? What happens as x goes to infinity?

Oh, and notice that the problem specifically asks you to "explain why they don't converge". Maybe the problem you are having isn't really a problem!
 
Last edited by a moderator:
mirandasatterley said:
Is there any way to combine the two so I can solve this.
Is there anything wrong with the most obvious approach: make both changes?
 
HallsofIvy said:
Well, the first thing you had better do is actually write out the anti- derivative!
What is \int \frac{1}{^3\sqrt{x}}dx= \int x^{-\frac{1}{3}}dx?

Does it converge as x goes to 0? What happens as x goes to infinity?

Oh, and notice that the problem specifically asks you to "explain why they don't converge". Maybe the problem you are having isn't really a problem!

The anti- derivative is X^(2/3)
2/3
As x goes to infinity, the anti-derivative goes to infinity.
As x goes to 0, the anti- derivative goes to 0.

so, would I evaluate it as (infinity - 0), which is infinity, therefore it diverges.

Is this right?
 
Question: A clock's minute hand has length 4 and its hour hand has length 3. What is the distance between the tips at the moment when it is increasing most rapidly?(Putnam Exam Question) Answer: Making assumption that both the hands moves at constant angular velocities, the answer is ## \sqrt{7} .## But don't you think this assumption is somewhat doubtful and wrong?

Similar threads

  • · Replies 7 ·
Replies
7
Views
2K
  • · Replies 1 ·
Replies
1
Views
2K
Replies
4
Views
2K
Replies
7
Views
2K
  • · Replies 2 ·
Replies
2
Views
2K
Replies
3
Views
2K
Replies
3
Views
2K
  • · Replies 105 ·
4
Replies
105
Views
6K
  • · Replies 6 ·
Replies
6
Views
2K
  • · Replies 13 ·
Replies
13
Views
3K